- PowerScore Staff
- Posts: 5972
- Joined: Mar 25, 2011
- Mon Jan 20, 2020 10:18 am
#88172
Complete Question Explanation
(The complete setup for this game can be found here: lsat/viewtopic.php?f=170&p=88166#p88166)
The correct answer choice is (A)
The question stem in this problem establishes that H receives two stars, and that H is the only CD to receive two stars. From the second rule we can determine that N receives just one star:
This information eliminates answer choice (A) from contention.
From the third rule, we can determine that I must receive the same rating as R. Pairing that inference with the deductions made in the discussion of the fourth rule, we can determine that I and R receive a rating of three stars, and that Q receives a rating of four stars:
Not surprisingly, S, the only random in the game, is not placed, although it must receive a rating of either one star or four stars.
As we have determined that I must receive a rating of three stars, answer choice (A) is proven correct.
Answer choices (B), (C), and (D) can never occur. Answer choice (E) is possible but does not have to occur.
(The complete setup for this game can be found here: lsat/viewtopic.php?f=170&p=88166#p88166)
The correct answer choice is (A)
The question stem in this problem establishes that H receives two stars, and that H is the only CD to receive two stars. From the second rule we can determine that N receives just one star:
This information eliminates answer choice (A) from contention.
From the third rule, we can determine that I must receive the same rating as R. Pairing that inference with the deductions made in the discussion of the fourth rule, we can determine that I and R receive a rating of three stars, and that Q receives a rating of four stars:
Not surprisingly, S, the only random in the game, is not placed, although it must receive a rating of either one star or four stars.
As we have determined that I must receive a rating of three stars, answer choice (A) is proven correct.
Answer choices (B), (C), and (D) can never occur. Answer choice (E) is possible but does not have to occur.
You do not have the required permissions to view the files attached to this post.
Dave Killoran
PowerScore Test Preparation
Follow me on X/Twitter at http://twitter.com/DaveKilloran
My LSAT Articles: http://blog.powerscore.com/lsat/author/dave-killoran
PowerScore Podcast: http://www.powerscore.com/lsat/podcast/
PowerScore Test Preparation
Follow me on X/Twitter at http://twitter.com/DaveKilloran
My LSAT Articles: http://blog.powerscore.com/lsat/author/dave-killoran
PowerScore Podcast: http://www.powerscore.com/lsat/podcast/